Difference between revisions of "2016 AMC 10A Problems/Problem 11"

(Solution 2)
(Solution 2)
Line 55: Line 55:
 
== Solution 2 ==
 
== Solution 2 ==
  
Find the area of the unshaded area by calculating the area of the triangles and rectangles outside of the shaded region. This will result in <math>40</math> - <math>33</math> <math>\frac{1}{2}</math> =  <math>\boxed{6\frac{1}{2}}</math>.
+
Find the area of the unshaded area by calculating the area of the triangles and rectangles outside of the shaded region. We can do this by splitting up the unshaded areas into various triangles and rectangles as shown.
 +
 
 +
<asy>
 +
 
 +
size(6cm);
 +
defaultpen(fontsize(9pt));
 +
draw((0,0)--(8,0)--(8,5)--(0,5)--cycle);
 +
filldraw((7,0)--(8,0)--(8,1)--(0,4)--(0,5)--(1,5)--cycle,gray(0.8));
 +
 
 +
label("$1$",(1/2,5),dir(90));
 +
label("$7$",(9/2,5),dir(90));
 +
 
 +
label("$1$",(8,1/2),dir(0));
 +
label("$4$",(8,3),dir(0));
 +
 
 +
label("$1$",(15/2,0),dir(270));
 +
label("$7$",(7/2,0),dir(270));
 +
 
 +
label("$1$",(0,9/2),dir(180));
 +
label("$4$",(0,2),dir(180));
 +
 
 +
draw((0,5/2)--(8,5/2));
 +
draw((4,0)--(4,5));
 +
 
 +
</asy>
 +
 
 +
Notice that the two added lines bisect each of the 4 sides of the large rectangle.
 +
 
 +
Subtracting the unshaded area from the total area gives us <math>40</math> - <math>33</math> <math>\frac{1}{2}</math> =  <math>\boxed{6\frac{1}{2}}</math>, so the correct answer is <math>\boxed{\textbf{(D)}}</math>.
  
 
==See Also==
 
==See Also==

Revision as of 20:03, 4 February 2016

Problem

What is the area of the shaded region of the given $8 \times 5$ rectangle?

[asy]  size(6cm); defaultpen(fontsize(9pt)); draw((0,0)--(8,0)--(8,5)--(0,5)--cycle); filldraw((7,0)--(8,0)--(8,1)--(0,4)--(0,5)--(1,5)--cycle,gray(0.8));  label("$1$",(1/2,5),dir(90)); label("$7$",(9/2,5),dir(90));  label("$1$",(8,1/2),dir(0)); label("$4$",(8,3),dir(0));  label("$1$",(15/2,0),dir(270)); label("$7$",(7/2,0),dir(270));  label("$1$",(0,9/2),dir(180)); label("$4$",(0,2),dir(180));  [/asy]

$\textbf{(A)}\ 4\dfrac{3}{5} \qquad \textbf{(B)}\ 5\qquad \textbf{(C)}\ 5\dfrac{1}{4} \qquad \textbf{(D)}\ 6\dfrac{1}{2} \qquad \textbf{(E)}\ 8$

Solution 1

First, split the rectangle into $4$ triangles: [asy]  size(6cm); defaultpen(fontsize(9pt)); draw((0,0)--(8,0)--(8,5)--(0,5)--cycle); filldraw((7,0)--(8,0)--(8,1)--(0,4)--(0,5)--(1,5)--cycle,gray(0.8));  label("$1$",(1/2,5),dir(90)); label("$7$",(9/2,5),dir(90));  label("$1$",(8,1/2),dir(0)); label("$4$",(8,3),dir(0));  label("$1$",(15/2,0),dir(270)); label("$7$",(7/2,0),dir(270));  label("$1$",(0,9/2),dir(180)); label("$4$",(0,2),dir(180));  draw((0,5)--(8,0));  [/asy]

The bases of these triangles are all $1$, and their heights are $4$, $\frac{5}{2}$, $4$, and $\frac{5}{2}$. Thus, their areas are $2$, $\frac{5}{4}$, $2$, and $\frac{5}{4}$, which add to the area of the shaded region, which is $\boxed{6\frac{1}{2}}$.

Solution 2

Find the area of the unshaded area by calculating the area of the triangles and rectangles outside of the shaded region. We can do this by splitting up the unshaded areas into various triangles and rectangles as shown.

[asy]  size(6cm); defaultpen(fontsize(9pt)); draw((0,0)--(8,0)--(8,5)--(0,5)--cycle); filldraw((7,0)--(8,0)--(8,1)--(0,4)--(0,5)--(1,5)--cycle,gray(0.8));  label("$1$",(1/2,5),dir(90)); label("$7$",(9/2,5),dir(90));  label("$1$",(8,1/2),dir(0)); label("$4$",(8,3),dir(0));  label("$1$",(15/2,0),dir(270)); label("$7$",(7/2,0),dir(270));  label("$1$",(0,9/2),dir(180)); label("$4$",(0,2),dir(180));  draw((0,5/2)--(8,5/2)); draw((4,0)--(4,5));  [/asy]

Notice that the two added lines bisect each of the 4 sides of the large rectangle.

Subtracting the unshaded area from the total area gives us $40$ - $33$ $\frac{1}{2}$ = $\boxed{6\frac{1}{2}}$, so the correct answer is $\boxed{\textbf{(D)}}$.

See Also

2016 AMC 10A (ProblemsAnswer KeyResources)
Preceded by
Problem 10
Followed by
Problem 12
1 2 3 4 5 6 7 8 9 10 11 12 13 14 15 16 17 18 19 20 21 22 23 24 25
All AMC 10 Problems and Solutions
2016 AMC 12A (ProblemsAnswer KeyResources)
Preceded by
Problem 7
Followed by
Problem 9
1 2 3 4 5 6 7 8 9 10 11 12 13 14 15 16 17 18 19 20 21 22 23 24 25
All AMC 12 Problems and Solutions

The problems on this page are copyrighted by the Mathematical Association of America's American Mathematics Competitions. AMC logo.png